LSAT and Law School Admissions Forum

Get expert LSAT preparation and law school admissions advice from PowerScore Test Preparation.

 Zarie Blackburn
PowerScore Staff
  • PowerScore Staff
  • Posts: 48
  • Joined: Jan 18, 2018
|
#77590
We recently received the following question from a student. An instructor will respond below. Thanks!
Hi. I know this is an older post but I wanted to ask a follow up question somewhat related to PT #71 Section 3 Question 11.
If a question asks you to chose the assumption that allows for the conclusion to be properly drawn- does that mean answers containing "even if" will never be correct for this question type, since "even if" statements do not affect the conditional reasoning?
This question was originally posted on the LSAT Blog here: https://blog.powerscore.com/lsat/how-to ... /#comments
 Jeremy Press
PowerScore Staff
  • PowerScore Staff
  • Posts: 1000
  • Joined: Jun 12, 2017
|
#77682
Hi!

Great question, and an important point you're raising. I'd encourage you to review again the blog post linked above, in particular the portion of that post that talks about the effect of an "even if" clause within a larger conditional statement. It says, in summary, "How does th[e] 'even if/ clause affect the conditional rule []? The answer, oddly enough, is that it doesn’t really affect it at all. This clause is simply stating that the conditional rule above holds true regardless of whether the ["even if" clause holds true]."

In this particular question, using the above insights, what we can do with the "even if" clause of answer choice B is disregard it. Now that, by itself, doesn't mean the answer choice is necessarily or automatically wrong. It just means that portion of answer choice B is extraneous. (Remember, extraneous information doesn't destroy an answer choice in a Justify question). Once we ignore that extraneous "even if" clause, we still have to look at the remaining conditional statement, which is, "[Secondary school students] will not achieve broad mastery of the curriculum if they do not devote significant efforts to their studies." And we have to ask whether that remaining conditional statement justifies the conclusion. If it did, it would be the correct answer! But it doesn't. And that's because the conclusion of the argument relies on adding some additional information to the contrapositive of the first sentence of the stimulus (the premise). But what answer choice B gives us is merely the Mistaken Negation of (a portion of) the first sentence of the stimulus. A Mistaken Negation will not justify a contrapositive (or, in this case, a souped-up contrapositive).

I hope this helps!

Jeremy

Get the most out of your LSAT Prep Plus subscription.

Analyze and track your performance with our Testing and Analytics Package.